Đến nội dung

the unknown

the unknown

Đăng ký: 10-03-2016
Offline Đăng nhập: 15-04-2023 - 15:30
****-

#648044 Trường hè Toán học Miền Nam.

Gửi bởi the unknown trong 05-08-2016 - 13:59

Ta viết lại đề như sau: Tìm số tự nhiên $N_0$ lớn nhất không thể biểu diễn được dưới dạng $6x+10y+15z$ trong đó $x,y,z$ là các số tự nhiên. Hay nói cách khác mọi số tự nhiên lớn hơn $N_0$ đều biểu diễn được dưới dạng $6x+15y+10z$ với $x,y,z$ là các số tự nhiên.

Để giải bài toán này, ta sử dụng bổ đề sau:

Định lí Sylvester: Cho $a,b$ là hai số nguyên dương nguyên tố cùng nhau. Khi đó $N_0=ab-a-b$ là số nguyên dương lớn nhất không biểu diễn được dưới dạng $ax+by$ với $x,y$ là các số tự nhiên.

Bằng định lí này ta sẽ chứng minh kết quả khác: Cho $a,b,c$ là ba số nguyên dương đôi một nguyên tố cùng nhau. Khi số $N_0=2abc-ab-bc-ca$ là số nguyên dương lớn nhất không thể biểu diễn được dưới dạng $abx+bcy+caz$ với $x,y,z$ là các số tự nhiên.

(1) Ta sẽ chứng minh $2abc-ab-bc-ca$ không biểu diễn được dưới dạng $abx+bcy+caz$. Thật vậy nếu tồn tại $x,y,z$ tự nhiên để:

$2abc-ab-bc-ca=abx+bcy+caz$

$\Leftrightarrow 2abc=ab(x+1)+bc(y+1)+ca(z+1)\Rightarrow b\mid ca(z+1)\Rightarrow b\mid z+1\Rightarrow z+1\geq b$

Tương tự ta được $x+1\geq c$ và $y+1\geq a$ và ta suy ra $ab(x+1)+bc(y+1)+ca(z+1)\geq 3abc>2abc$ vô lí.

(2) Ta sẽ chứng minh tồn tại $x,y,z$ để $m+2abc-ab-bc-ca+1=abx+bcy+caz$ với $m\geq 0$. Biến đổi đẳng thức tương đương với:

$b(ax+cy-ac+a+c-1)+acz=m+abc-ac-b+1\geq abc-ac-b+1$

Theo định lí Sylvester thì tồn tại $u,z$ tự nhiên để $bu+acz=m+abc-ac-b+1$. Hơn nữa do $u\geq 0$ nên cũng tồn tại $x,y$ tự nhiên để $ax+cy=u+ac-a-c+1$. Và từ đó dễ dàng suy ra tồn tại $x,y,z$ tự nhiên thỏa mãn. Vậy kết quả được chứng minh.

Với $a=2,b=3,c=5$ thì ta được $N_0=29$. Vậy $29$ là số lớn nhất thỏa mãn đề bài.




#646983 Marathon Phương trình và hệ phương trình VMF

Gửi bởi the unknown trong 29-07-2016 - 08:45

Bạn thử kiểm tra lại.  Chú ý số hạng $\dfrac{x^{2}+x\sqrt{x}+2}{x+x\sqrt{x}+4}.$

 

Phương trình này có ít nhất một nghiệm khác trong $\left(\frac{3}{2},2\right)$?

À vâng em có đã kiểm tra và hơi sai sót một tí. Thật sự ra thì cách giải của em không có vấn đề gì mà do anh NTA1907 ghi đề bị sai sót đó anh. Đề chính xác phải là:

$\dfrac{3+\sqrt{x}}{x^{2}+x\sqrt{x}+x+3}+\dfrac{x+\sqrt{x}+2}{x^{2}+x\sqrt{x}+4}+\dfrac{x\sqrt{x}+x+2}{x^{2}+\sqrt{x}+4}+\dfrac{x^{2}+x\sqrt{x}+2}{x+\sqrt{x}+4}+\dfrac{x^{2}+3}{x\sqrt{x}+x+\sqrt{x}+3}=\dfrac{10}{3}$

Tức là phải sửa số hạng $\frac{x^2+x\sqrt{x}+2}{x+x\sqrt{x}+4}$ thành $\frac{x^2+x\sqrt{x}+2}{x+\sqrt{x}+4}$.

Đây hình như là đề đề nghị Olympic 30/4.

Nhân tiện vì không có ai gửi bài nên em xin tiếp tục:

Bài 93: Giải phương trình: $\sqrt{2x+15}=32x^2+32x-20$.




#646935 Marathon Phương trình và hệ phương trình VMF

Gửi bởi the unknown trong 28-07-2016 - 20:32

Bài 91: Giải phương trình:

$\dfrac{3+\sqrt{x}}{x^{2}+x\sqrt{x}+x+3}+\dfrac{x+\sqrt{x}+2}{x^{2}+x\sqrt{x}+4}+\dfrac{x\sqrt{x}+x+2}{x^{2}+\sqrt{x}+4}+\dfrac{x^{2}+x\sqrt{x}+2}{x+x\sqrt{x}+4}+\dfrac{x^{2}+3}{x\sqrt{x}+x+\sqrt{x}+3}=\dfrac{10}{3}$

Bài này thì ý tưởng là dùng bất đẳng thức:

Điều kiện $x\geq 0$

Ta đặt $a=2,b=\sqrt{x}+1,c=x^2+1,d=x\sqrt{x}+1,e=x+1$. Khi đó phương trình tương đương: $\sum \frac{a+b}{c+d+e}\geq \frac{10}{3}$

Ta sẽ chứng minh: $\sum \frac{a+b}{c+d+e}\geq \frac{10}{3}$ với mọi $a,b,c,d,e$ dương.

Thật vậy ta có: $\sum \frac{a+b}{c+d+e}\geq \frac{10}{3}\Leftrightarrow (a+b+c+d+e)\sum \frac{1}{a+b+c}\geq \frac{25}{3}$.

Mà theo BĐT Cauchy Schwarz thì $\sum \frac{1}{a+b+c}\geq \frac{25}{3(a+b+c+d+e)}$ 

Tù đó ta suy ra điều phải chứng minh. Hơn nữa đẳng thức xảy ra khi và chỉ khi $a=b=c=d=e$ tức là $x=1$. Từ đó suy ra phương trình có nghiệm duy nhất $x=1$.

 

Mạn phép cho em nói một tí  :D : Em thấy topic có vẻ hơi lạc hướng của Marathon mà dần trở thành Topic thảo luận về phương trình và hệ phương trình rồi ạ.

 

Vì hướng tới thi olympic nên em xin đề xuất bài tiếp theo:

Bài 92: Giải hệ phương trình:

$\left\{\begin{matrix} x_1+x_2+x_3+x_4+x_5=1\\ (x_1+x_2)(x_1+x_2+x_3)(x_1+x_2+x_3+x_4)=256x_1x_2x_3x_4x_5\\ x_1,x_2,x_3,x_4,x_5> 0\\ \end{matrix}\right.$




#646781 Chứng minh rằng $a<b$

Gửi bởi the unknown trong 27-07-2016 - 20:01

Giả sử $a$ là nghiệm của phương trình:

$2013x^3+2014x^2+2015x+2016=0$

Và $b$ là nghiệm của phương trình:

$2014x^3+2015x^2+2016x+2017=0$

Chứng minh rằng $a<b$.




#646745 Chứng minh $N> M$

Gửi bởi the unknown trong 27-07-2016 - 16:23

Gọi $N$ là số nghiệm nguyên của phương trình $x^2-y^2=z^3-t^3$, với điều kiện $0\leq x, y, z, t\leq 10^6$, và $M$ là số nghiệm nguyên của phương trình $x^2-y^2=z^3-t^3+1$, với điều kiện tương tự. Chứng minh $N> M$.

(IMO Shortlist 1979)

Hai phương trình trên tương đương: $x^2+t^3=y^2+z^3$ và $x^2+t^3=y^2+z^3+1$.

Ta kí hiệu $f(n)$ tương ứng là số nghiệm của phương trình $a^2+b^3=n$ với $n$ là số tự nhiên.

Ta xét với một số tự nhiên $n$ $(0\leq n\leq 10^{12}+10^{18})$ thì ta có phương trình: $x^2+t^2=y^2+z^3=n$ thì khi đó phương trình $x^2+t^3=n$ có $f(n)$ nghiệm và phương trình $y^2+z^3=n$ cũng có $f(n)$ nghiệm. Vậy khi đó số nghiệm $(x,y,z,t)$ của phương trình $x^2+t^3=y^2+z^3=n$ là $f(n)^2$. Và như thế, đặt $k=10^{12}+10^{18}$ thì ta sẽ tính được: 

$N=\sum _{n=0}^k f(n)^2$

Tương tự: Ta xét với một số tự nhiên $n$ $(0\leq n\leq 10^{12}+10^{18})$ thì ta có phương trình: $x^2+t^2=y^2+z^3+1=n$. Khi đó phương trình $x^2+t^3=n$ có $f(n)$ nghiệm còn phương trình $y^2+z^3+1=n$ có $f(n-1)$ nghiệm nên

$M=\sum _{n=0}^k f(n).f(n-1)$

Ta có:

$N-M=\sum _{n=0}^k f(n)^2-\sum _{n=0}^k f( n).f(n-1)=\frac{1}{2} \sum _{n=0}^k(f(n)-f(n-1))^2> 0$

Bởi vì đẳng thức không thể xảy ra do $f(0)\neq f(1)$. (do $f(0)=1, f(1)=2$)

Nên $N>M$ và ta kết thúc chứng minh.




#646461 $a, b, c > 0.$ Chứng minh: $\sum \frac{a^3}{a^2+...

Gửi bởi the unknown trong 25-07-2016 - 20:06

Bài 2: a, b, c > 0. ab + bc + ca = 1. CMR:

$a^{2}+b^{2}+c^{2}+\frac{8abc}{(a+b)(b+c)(c+a)} \geq 2$

 

Bài 4: a, b, c > 0. CMR:

$\frac{a^{2}+b^{2}+c^{2}}{ab+bc+ca} + \frac{8abc}{(a+b)(b+c)(c+a)}\geq 2$

Về cơ bản hai bài này là giống nhau nên mình gộp chung:

Không mất tính tổng quát giả sử $c$ là số nhỏ nhất.

Ta có:

$\frac{a^2+b^2+c^2}{ab+bc+ca}+\frac{8abc}{(a+b)(b+c)(c+a)}\geq 2$

$\Leftrightarrow M(a-b)^2+N(a-c)(b-c)\geq 0$

Trong đó:

$M=\frac{1}{ab+bc+ca}-\frac{2c}{(a+b)(b+c)(c+a)}$

$N=\frac{1}{ab+bc+ca}-\frac{1}{(b+c)(c+a)}$

Dễ chứng minh được:

$N=\frac{c^2}{(b+c)(c+a)(ab+bc+ca)}\geq 0$

$M=\frac{ab(a+b)+bc(b-c)+ca(a-c)}{(ab+bc+ca)(a+b)(b+c)(c+a)}\geq 0$

Vậy $M,N\geq 0$ nên bất đẳng thức đúng. Vậy ta có điều phải chứng minh. Đẳng thức xảy ra khi $a=b=c$. ( Nếu điều kiện là $a,b,c$ không âm thì bài toán còn xảy ra đẳng thức tại $a=b$ và $c=0$ nữa)




#646398 Tìm tất cả các số nguyên dương giảm.

Gửi bởi the unknown trong 25-07-2016 - 10:49

Một số nguyên dương $a$ được gọi là giảm về số nguyên dương $b$ nếu ta chia $a$ cho chữ số hàng đơn vị của nó thì ta được số $b$. ( Ví dụ như $2016$ giảm về $\frac{2016}{6}=336$)

Hãy tìm tất cả các số nguyên dương có thể giảm về số $1$ sau một số lần giảm. Ví dụ như $12$ là một số có tính chất như vậy vì $12$ giảm về $6$ và $6$ giảm về $1$.

(Mexico 2014)

 

P.s




#645917 $\sum_{k=1}^n \frac{f(k)}{k^2}...

Gửi bởi the unknown trong 21-07-2016 - 20:25

 

Cho $n \in \mathbb{N^*}$ và $f : \{1,2,..,n\} \rightarrow \{1,2..,n\}$ là một song ánh . Chứng minh rằng : 
$\sum_{k=1}^n \frac{f(k)}{k^2} \ge \sum_{k=1}^n \frac{1}{k}$

 

Bài này nếu dùng bất đẳng thức hoán vị thì nhanh rồi nhỉ  :)

Nhắc lại về bất đẳng thức hoán vị: Cho hai dãy số thực $(a_1,a_2,...,a_n)$ và  $(b_1,b_2,...,b_n)$ cùng tăng hoặc cùng giảm. Khi đó giả sử $(b_{k_1},b_{k_2},...,b_{k_n})$ là một hoán vị của $(b_1,b_2,...,b_n)$ thì ta có bất đẳng thức: $a_1b_1+a_2b_2+...+a_nb_n\geq a_1b_{k_1}+a_2b_{k_2}+...+a_nb_{k_n}\geq a_1b_n+a_2b_{n-1}+...+a_nb_1$.

Như vậy nhận xét rằng $(1,2,...,n)$ và $(\frac{1}{n^2},\frac{1}{(n-1)^2},...,1)$ là hai dãy cùng tăng. Hơn nữa ánh xạ $f: \{1,2,...,n\} \rightarrow \{1,2,...,n\}$ là một song ánh nên $(f(1),f(2),...,f(n))$ là một hoán vị của $(1,2,...,n)$. Từ đó theo bất đẳng thức hoán vị ta được: $\sum_{k=1}^n \frac{f(k)}{k^2} \ge \sum_{k=1}^n \frac{1}{k}$.

Vậy bài toán được chứng minh.

 

P.s: Hóng cách giải hay hơn. Chứ dùng bất đẳng thức hoán vị thì ác quá.   :D




#645773 ĐỀ THI OLYMPIC GẶP GỠ TOÁN HỌC LẦN VIII

Gửi bởi the unknown trong 21-07-2016 - 08:31

Khối 10 bài 3 ( bất đẳng thức): Cho $\Delta ABC$ có độ dài các cạnh không vượt quá $1$. Gọi $p,R,r$ lần lượt là nửa chu vi, bán kính đường tròn ngoại tiếp và nội tiếp tam giác. Chứng minh rằng:

$p(1-2Rr)\leq 1$

 

Kí hiệu $a,b,c$ là các cạnh của tam giác. Ta sử dụng các hệ thức lượng quen thuộc: $S=\frac{abc}{4R},S=pr$. Từ đó ta có:

$p(1-2Rr)=p-2Rpr=p-2RS=p-\frac{abc}{2}=\frac{a+b+c-abc}{2}$.

Do đó bất đẳng thức được chứng minh nếu ta chứng minh được: $a+b+c-abc\leq 2$ với $0<a,b,c\leq 1$ và $a,b,c$ là các cạnh của tam giác.

Do $0<c\leq 1$ và $1-ab\leq 0$ nên $a+b+c-abc=a+b+c(1-ab)\leq a+b+1-ab$

Do đó ta chứng minh $a+b+1-ab\leq 2$ tương đương $(a-1)(b-1)\geq 0$ (luôn đúng do $0<a,b,c\leq 1$)

Vậy bài toán được chứng minh. Đẳng thức xảy ra khi $a=b=c=1$, tức là $\Delta ABC$ đều và có các cạnh bằng $1$.

 

P.s: Bài 2 lớp 10 em có ý tưởng là dùng quy nạp để chứng minh. Tuy nhiên lập luận hơi lỏng lẻo nên chưa dám đăng  :D




#645408 $n=\pm 1^2\pm 2^2\pm \cdots \pm k^2$

Gửi bởi the unknown trong 18-07-2016 - 15:24

(Olympic GGTH 2016) Chứng minh rằng với mỗi số nguyên $n$ thì tồn tại vô hạn cách để biểu diễn $n$ dưới dạng:

$n=\pm 1^2\pm 2^2\pm \cdots \pm k^2$

với số nguyên dương $k$ và các dấu $+,-$ được chọn phù hợp.




#645367 CMR: Trong dãy có số các số 2 nhiều hơn số các số 1

Gửi bởi the unknown trong 18-07-2016 - 08:06

gọi các số trong dãy là $2^{a}$ với$1\leq a\leq 2005$

xét a lẻ ta có $2^{a}=2^{2k+1} \equiv 2^{2k}.2\equiv 4^{k}.2\equiv 2 (mod 3)$ tổng các chữ số sau cùng của các số này là 2

xét a chẵn thì $2^{2k}\equiv 1 (mod 3)$ tổng các chữ số sau cùng của các chữ số này là 1 
suy ra điều phải chứng minh 
p/s: không biết em làm vậy có chặt chẽ hay không nữa :))

Theo mình nghĩ là thế này: Nếu bạn xét modulo $3$ thì theo mình nghĩ điều đó là chưa đủ, chẳng hạn như nếu một số chia $3$ dư $1$ thì nếu ta thực hiên như đề bài ta sẽ quy về một số có $1$ chữ số chia $3$ dư $1$ như cách của bạn ( phần này đúng không có vấn đề gì  ^_^ ). Nhưng vấn đề là ngoài số $1$ còn các số $4$ và $7$ nữa. Nên nếu xét như bạn thì ta không thể khẳng định số số $2$ nhiều hơn số số $1$. Nếu muốn chắc chắn hơn thì cách tốt hơn là xét modulo $9$ ( dài hơn xíu nhưng cũng không có vấn đề gì  ^_^ )

Mạn phép giải luôn  ^_^ : Dễ chứng minh được $2^a$ chia $9$ dư $1$ khi và chỉ khi $a=6k$ và $2^a$ chia $9$ dư $2$ khi và chỉ khi $a=6k+1$. Do đó bằng cách đếm các số chia hết cho $6$ và chia $6$ dư $1$ ta dễ thấy số các số chia $9$ dư $2$ nhiều hơn số các số chia $9$ dư $1$ nên lập luận tiếp ta suy ra điều phải chứng minh.




#645366 Inequalities From 2016 Mathematical Olympiads

Gửi bởi the unknown trong 18-07-2016 - 07:51

Bài 49 (Bulgaria). Cho $a,b,c,d0$ là bốn số thực dương. Chứng minh rằng

\[\frac {a+\sqrt{ab}+\sqrt[3]{abc}+\sqrt[4]{abcd}}{4} \leq \sqrt[4]{a \cdot \frac{a+b}{2} \cdot \frac{a+b+c}{3} \cdot \frac{a+b+c+d}{4}}.\]
 

Ta có các BĐT sau theo AM-GM cho 4 số thực dương:

$\frac{1}{4}(1+\frac{2a}{a+b}+\frac{3a}{a+b+c}+\frac{4a}{a+b+c+d})\geq \sqrt[4]{\frac{2a}{a+b}.\frac{3a}{a+b+c}.\frac{4a}{a+b+c+d}}$

 

$\frac{1}{4}\left ( 1+1+\frac{2b}{a+b}+\frac{4c}{a+b+c+d} \right )\geq \sqrt[4]{\frac{2b}{a+b}.\frac{4c}{a+b+c+d}}$

 

$\frac{1}{4}\left ( 1+\frac{2b}{a+b}+\frac{3c}{a+b+c}+\frac{4d}{a+b+c+d}\right )\geq \sqrt[4]{\frac{2b}{a+b}.\frac{3c}{a+b+c}.\frac{4d}{a+b+c+d}}$

 

$\frac{1}{4}(1+\frac{2a}{a+b}+\frac{3b}{a+b+c}+\frac{4b}{a+b+c+d})\geq \sqrt[4]{\frac{2a}{a+b}.\frac{3b}{a+b+c}.\frac{4b}{a+b+c+d}}$

 

Cộng tất cả các bất đẳng thức trên lại và ta thu được:

 

$4\geq \sqrt[4]{\frac{2a}{a+b}.\frac{3a}{a+b+c}.\frac{4a}{a+b+c+d}}+\sqrt[4]{\frac{2b}{a+b}.\frac{4c}{a+b+c+d}}+\sqrt[4]{\frac{2b}{a+b}.\frac{3c}{a+b+c}.\frac{4d}{a+b+c+d}}+\sqrt[4]{\frac{2a}{a+b}.\frac{3b}{a+b+c}.\frac{4b}{a+b+c+d}}$

 

Suy ra:

 

$\sqrt[4]{a.\frac{a+b}{2}.\frac{a+b+c}{3}.\frac{a+b+c+d}{4}}\geq \frac{1}{4}(a+\sqrt{ab}+\sqrt[4]{abc.\frac{a+b+c}{3}}+\sqrt[4]{abcd})\geq \frac{1}{4}(a+\sqrt{ab}+\sqrt[4]{abc.\sqrt[3]{abc}}+\sqrt[4]{abcd})=\frac{a+\sqrt{ab}+\sqrt[3]{abc}+\sqrt[4]{abcd}}{4}$

Vậy bài toán được chứng minh. Đẳng thức xảy ra khi và chỉ khi $a=b=c=d$.

 

Remark




#644849 Inequalities From 2016 Mathematical Olympiads

Gửi bởi the unknown trong 13-07-2016 - 21:26

Bài 48 (Japan MO Final). Cho bốn số thực $a,b,c,d$ thỏa mãn điều kiện $ab+bc+cd =1.$
Tìm giá trị nhỏ nhất của biểu thức $$P (a^2+ac+c^2)(b^2+bd+d^2).$$
 

Bài 48: Ta xét: $S=(a^2+ac+c^2)(b^2+bd+d^2)-(ab+bc+cd)^2=(ad-bc)(ab+ad+cd)=(ad-bc)(1+ad-bc)=(ad-bc+\frac{1}{2})^2-\frac{1}{4}\geq -\frac{1}{4}$

$\Rightarrow P\geq (ab+bc+cd)^2-\frac{1}{4}=\frac{3}{4}$

Đẳng thức xảy ra tương đương với việc ta có hệ: $\left\{\begin{matrix} ad-bc=-\frac{1}{2}\\ ab+bc+cd=1\\ \end{matrix}\right.$

Hệ này có nghiệm, ví dụ như bộ nghiệm: $(a,b,c,d)\sim (0,\frac{1}{2},1,\frac{1}{2})$

Như vậy ta có giá trị nhỏ nhất của $P$ là $\frac{3}{4}$, xảy ra chẳng hạn khi $(a,b,c,d)\sim (0,\frac{1}{2},1,\frac{1}{2})$.




#644671 Thảo luận về Đề thi và Lời giải của IMO 2016

Gửi bởi the unknown trong 12-07-2016 - 15:53

 

attachicon.gifIMO.png

 

 

Bài 3. Cho $P=A_1A_2\ldots A_k$ là một đa giác lồi trong mặt phẳng. Các đỉnh $A_1,A_2,\ldots A_k$ có tọa độ là các số nguyên và nằm trên một đường tròn. Gọi $S$ là diện tích của $P$. Một số tự nhiên $n$ lẻ thỏa mãn bình phương độ dài các cạnh của $P$ đều chia hết cho $n$. Chứng minh rằng $2S$ là một số tự nhiên chia hết cho $n$

 

Đây là một lời giải em tham khảo ở AoPS cho bài số 3, bài được cho là khó nhất của IMO năm nay  :)

Bài 3: Trước hết, giả sử rằng đa giác $A_1A_2\ldots A_k$ có một đường chéo thỏa mãn bình phương của đường chéo đó là một số nguyên chia hết cho $n$, khi đó ta sẽ cắt đa giác này bằng đường chéo này thành hai đa giác với số cạnh nhỏ hơn. Do đó không giảm tổng quát có thể giả sử đa giác $A_1A_2\ldots A_k$ không có một đường chéo nào có bình phương chia hết cho $n$.

 

Giả sử $k\ge 4$. Không mất tính tổng quát, ta sẽ chứng minh cho trường hợp $n=p^a$ ($p$ là số nguyên tố lẻ). Khi đó, xét một số tự nhiên $r$ thỏa $a>r\geq 0$ và $p^r\mid A_iA_j^2$ với mọi $i\neq j$, $0<i,j\le k$. Hiển nhiên ta có $r+1\leq a$ nên $p^{r+1}\mid p^a\mid A_iA_j^2$ nếu $A_i,A_j$ là hai đỉnh kề nhau.

 

Ta sẽ chứng minh bằng quy nạp rằng nếu $r$ là một số tự nhiên thỏa mãn điều kiện trên thì $r+1$ cũng sẽ thỏa mãn các điều kiện trên. Do tất cả các đỉnh của đa giác cùng nằm trên một đường tròn nên xét cụ thể cho một tứ giác $A_{i-1}A_iA_{i+1}A_j$ và cho đơn giản, ta đặt: $A_{i-1}A_i=b,A_{i}A_{i+1}=c,A_{i+1}A_j=d,A_jA_{i-1}=e,A_{i-1}A_{i+1}=f,A_iA_j=g$.

 

Và theo giả thiết quy nạp, ta đặt: $b^2=p^ax,c^2=p^ay,d^2=p^rz,e^2=p^rk,f^2=p^rw,g^2=p^rt$ ( với $x,y,z,k,w,t$ là các số nguyên) Khi đó theo định lý Ptolemy thì ta có được:$bd+ce=fg \Rightarrow f^2g^2=b^2d^2+c^2e^2+2bcde=p^{a+r}(xz+yk+2\sqrt{xyzk})$. Từ đó ta có $\sqrt{xyzk}$ là một số hữu tỉ nên cũng là một số nguyên. Do đó ta có được: $p^{a+r}\mid f^2g^2\Rightarrow v_{p}(f^2g^2)\geq a+r$. Tuy nhiên: vì $A_iA_j$ và $A_{i-1}A_{i+1}$ là các đường chéo nên $v_{p}(f^2)\leq a-1\Rightarrow v_{p}(g^2)\geq r+1\Rightarrow p^{r+1}\mid g^2$

 

Tức là khi đó $p^{r+1}\mid A_iA_j^2$ với $A_iA_j$ là một đường chéo và tương tự với các đường chéo khác, ta có được $p^{r+1}\mid A_iA_j^2$ với mọi $i\neq j$, $0<i,j\leq k$. Như vậy theo nguyên lí quy nạp, giả thiết được chứng minh. Khi đó theo giả thiết thì $p^{a-1}\mid A_iA_j^2$ với mọi $i\neq j$ và cũng chứng minh tương tự mà $p^a\mid A_iA_j^2$ với mọi $i\neq j$. Đây là điều vô lí ( do ta đã giả sử rằng không có đường chéo nào chia hết cho $p^a$).

 

Do đó $k<4$, tức là ta sẽ quy về trường hợp đơn giản $k=3$. Ta có thể chứng minh điều này cách dễ dàng vì các đỉnh $A_1,A_2,A_3$ có tọa độ nguyên nên dễ dàng chứng minh được $2S$ là số nguyên.

Hơn nữa theo công thức Hê-rông thì $2S=\frac{\sqrt{4a^2b^2-(a^2+b^2-c^2)^2}}{2}$ ($a,b,c$ là các cạnh của tam giác) nên $4.(2S)^2=(4a^2b^2-(a^2+b^2-c^2)^2)\vdots n^2$. Do $n$ lẻ nên $(4,n)=1$ do đó $n^2\mid (2S)^2$ nên $n\mid 2S$.

Vậy bài toán được chứng minh hoàn toàn.$\blacksquare$

 

P.s: Hình như ngày 2 không có hình  :(




#644460 MAX: $T=\frac{a(b+c)}{(b+c)^2+a^2}+\frac...

Gửi bởi the unknown trong 11-07-2016 - 09:14

Cho a,b,c dương. Tìm GTLN của:

$T=\frac{a(b+c)}{(b+c)^2+a^2}+\frac{b(c+a)}{(c+a)^2+b^2}+\frac{c(a+b)}{(a+b)^2+c^2}$

Chuẩn hóa $a+b+c=3$. Khi đó ta sẽ tìm GTLN của: $\sum \frac{a(3-a)}{(3-a)^2+a^2}$.

Ta sẽ chứng minh: $\frac{a(3-a)}{(3-a)^2+a^2}\leq \frac{9}{25}a+\frac{1}{25}\Leftrightarrow \frac{9(a-1)^2(2a+1)}{25(a^2+(3-a)^2)}\geq 0$ ( hiển nhiên đúng)

Do đó $\sum \frac{a(3-a)}{a^2+(3-a)^2}\leq \sum (\frac{9}{25}a+\frac{1}{25})= \frac{6}{5}$

Nên GTLN của $T$ là  $\frac{6}{5}$, xảy ra khi $a=b=c$.